UPSC Prelims 2023 Detailed Solutions

Detailed Prelims 2023 solution with reference to authentic sources. Thoroughly cross-checked and refined.


Table of contents

Estimating Difficulty Levels

Easy - given directly in basic books or NCERTs (34 Q)

Medium - concept given in basic books needs to be applied (36 Q)

Hard - Worth skipping in actual paper (30 Q)

If you're only interested in the Answer Key, go here.


Prelims 2023 Cutoff Prediction

We have estimated a cutoff based on two methods.

  1. Difficulty level assessment of the GS paper.
  2. Data collected from social media handles via this form.

These two methods would serve to cross verify our finding so that we can give  a more robust prediction. Further, we have also adjusted the cutoff keeping in mind the difficulty of the CSAT exam, which we feel will push the cutoff lower.

Method 1 - Estimating Difficulty

Easy Medium Hard
Questions 34 36 30
Accuracy 0.8 0.6 0.3
Positive 54.4 43.2 18
Negative -4.5 -9.6 -4
Total 49.9 33.6 4

Total Marks = 87.5

But this is when you're attempting all 100Qs. So cutoff, once we adjust for number of questions attempted and difficulty of CSAT falls below this.

Cutoff Predicted: 80-85 Marks.

Method 2 - Form Data

Entries received: 3052

(About 900 entries were removed from the data. Reasons: failing in CSAT, assumed 50% success rate for border case candidates, and bogus entries)

Mains candidates: 12,000

People sitting for Prelims: 5,00,000

Cutoff should be at the 97.6th percentile score from the data collected.

Cutoff Predicted - 82-84 Marks.


A Note on Controversial Questions

We have highlighted most of the questions where disagreements exist in the answer keys of various institutes. You can simply ctrl+F "controversial question" and you will find them.


Detailed Solutions

Q1. In which one of the following regions was Dhanyakataka, which flourished as a prominent Buddhist centre under the Mahasanghikas, located?

(a) Andhra

(b) Gandhara

(c) Kalinga

(d) Magadha

Ans: A

Difficulty: Medium

Subject: Ancient/Medieval/Art and Culture

Solution: Dhanyakataka is the old name of Amravati (capital of Andhra Pradesh). Dharanikota is a town near Amaravati in the Guntur district of Andhra Pradesh in India, It is the site of the ancient Dhanyakataka which was the capital of the Satavahana kingdom, also known as Andhras which ruled in the Deccan around the 1st to 3rd centuries A.D.

(https://frontline.thehindu.com/other/advertorial/article30207957.ece)

Source: https://www.wisdomlib.org/definition/dhanyakataka


Q2. With reference to ancient India, consider the following statements:

  1. The concept of Stupa is Buddhist in origin.
  2. Stupa was generally a repository of relics.
  3. Stupa was a votive and commemorative structure in Buddhist tradition

How many of the statements given above are correct?
(a) Only one
(b) Only two
(c) All three
(d) None

Ans: B

Difficulty: Easy

Subject: Ancient/Medieval/Art and Culture

Solution: 2 and 3 are correct. Stupa predates Buddhism. So statement 1 is incorrect.

Source: https://www.khanacademy.org/humanities/ap-art-history/introduction-cultures-religions-apah/buddhism-apah/a/the-stupa



Q3. With reference to ancient South India, Korkai, Poompuhar and Muchiri were well known as
(a) capital cities
(b) ports
(c) centres of iron-and-steel making
(d) shrines of Jain Tirthankaras

Ans: B

Difficulty: Hard (Current Affairs)

Subject: Ancient/Medieval/Art and Culture

Solution: These were ancient port cities.

Source: https://www.thehindu.com/news/cities/Madurai/excavation-at-ancient-port-city-of-korkai-begins/article65845179.ece


Q4. Which one of the following explains the practice of 'Vattakirutal' as mentioned in Sangam poems?
(a) Kings employing women bodyguards
(b) Learned persons assembling in royal courts to discuss religious and philosophical matters
(c) Young girls keeping watch over agricultural fields and driving away birds and animals
(d) A king defeated in a battle committing ritual suicide by starving himself to death

Ans: D

Difficulty: Hard

Subject: Ancient/Medieval/Art and Culture

Solution:

Screenshot from book: A History of Ancient and Early Medieval India by Upinder Singh


Q5. Consider the following dynasties

  1. Hoysala
  2. Gahadavala
  3. Kakatiya
  4. Yadava

How many of the above dynasties established their kingdoms in early eighth century AD?
(a) Only one
(b) Only two
(c) Only three
(d) None

Ans: D

Difficulty: Medium

Subject: Ancient/Medieval/Art and Culture

Solution: All of the above dynasties were established after the eighth century.

These dynasties were established in:

1. Hoysala dynasty - 10th Century AD (https://www.britannica.com/topic/Hoysala-dynasty)

2. Gahadavala dynasty - 11th Century AD (https://www.britannica.com/topic/Gahadavala-dynasty)

3. Kakatiya dynasty - 12th Century AD (https://en.wikipedia.org/wiki/Kakatiya_dynasty)

4. Yadava dynasty - 12th Century AD   (https://www.britannica.com/topic/Yadava-dynasty)



Q6. With reference to ancient Indian History, consider the following pairs:
Literary work      Author
1. Devichandraguptam - Bilhana
2. Hammira-Mahakavya - Nayachandra Suri
3. Milinda-panha    - Nagarjuna
4. Nitivakyamrita   - Somadeva Suri
How many of the above pairs are correctly matched?
(a) Only one
(b) Only two
(c) Only three
(d) All four

Ans: B/C

Difficulty: Hard

Subject: Ancient/Medieval/Art and Culture

Solution: 2 and 4 are correct. Correct pairs are:

Hammira-Mahakavya - Nayachandra Suri
Nitivakyamrita    - Somadeva Suri

Controversial Question: There is confusion regarding the third pair (Milinda-panha - Nagarjuna). Some scholars believe that Nagasena (who wrote Milindpanho) and Nagarjuna were the same person - in that case the answer will be C.


Q7. "Souls are not only the property of animal and plant life, but also of rocks, running water and many other natural objects not looked on as living by other religious sects."
The Above statement reflects one of the core beliefs of which one of the following religious sects of ancient India?
(a) Buddhism
(b) Jainism
(c) Shaivism
(d) Vaishnavism

Ans: B

Difficulty: Easy

Subject: Ancient/Medieval/Art and Culture

Solution:

Source: https://education.nationalgeographic.org/resource/jainism/#


Q8. Who among Vijayanagara the following rulers of Empire constructed a large dam across Tungabhadra River and a canal-cum-aqueduct several kilometres long from the river to the capital city?
(a) Devarava I
(b) Mallikarjuna
(c) Vira Vijaya
(d) Virupaksha

Ans: A

Difficulty: Hard

Subject: Ancient/Medieval/Art and Culture

Solution:

Source: V.K Agnihotri (2007), India History, p.150


Q9. Who among the following rulers of medieval Guiarat surrendered Diu to the Portuguese?
(a) Ahmad Shah
(b) Mahmud Begarha
(c) Bahadur Shah
(d) Muhammad Shah

Ans: C

Difficulty: Easy

Subject: Ancient/Medieval/Art and Culture

Solution: Bahadur Shah surrendered Diu to the Portuguese.

Source: Spectrum by Rajiv Ahir : Modern Indian History (Chapter: Advent of the Europeans in India, Section: Portuguese in India)


Q10. By which one of the following Acts was the Governor General of Bengal designated as the Governor General of India?
(a) The Regulating Act
(b) The Pitt's India Act
(c) The Charter Act of 1793
(d) The Charter Act of 1833

Ans: D

Difficulty: Easy

Subject: Modern Indian History

Solution:

Source: IGNOU- https://egyankosh.ac.in/bitstream/123456789/20342/1/Unit-23.pdf Page no. 10



Q11. In essence, what does 'Due Process of Law' mean?
(a) The principle of natural justice
(b) The procedure established by law
(c) Fair application of law
(d) Equality before law

Ans: A/C

Difficulty: Medium

Subject: Polity

Solution:

Controversial Question: Natural justice also includes fair and just process of law. So, this question is open to interpretation.

Source: https://www.investopedia.com/terms/d/due-process.asp



Q12. Consider the following statements:
Statement-I: In India, prisons are managed by State Governments with their own rules and regulations for the day-to-day administration of prisons.
Statement-II: In India, prisons are governed by the Prisons Act, 1894 which expressly kept the subject of prisons in the control of Provincial Governments.
(a) Both Statement-I and Statement-II are correct and Statement-II is the correct explanation for Statement-I
(b) Both Statement-I and Statement-II are correct and Statement-II is not the correct explanation for Statement-I
(c) Statement-I is correct but Statement-II is incorrect
(d) Statement-I is incorrect but Statement-II is correct

Ans: C

Difficulty: Medium

Subject: Polity

Solution: Statement I is correct: As per Schedule 7 of Indian Constitution, Prison is a State subject - entry 4)

Statement II is not correct: 1935 ACT PUT PRISONS AS PROVINCIAL SUBJECT

Source: https://www.mea.gov.in/Images/pdf1/S7.pdf



Q13. Which one of the following statements best reflects the Chief purpose of the 'Constitution' of a country?
(a) It determines the objective for the making of necessary laws.
(b) It enables the creation of political offices and a government.
(c) It defines and limits the powers of government.
(d) It secures social justice, social equality and social security.

Ans: C

Difficulty: Easy

Subject: Polity

Solution: Direction question. Similar question was asked in previous prelims.

Source: PYQs,

https://onlinelibrary.wiley.com/doi/abs/10.1111/j.1070-3535.2006.00447.x#:~:text=Constitutional%20government%20is%20by%20definition,by%20separating%20or%20dividing%20it.



Q14. In India, which one of the following Constitutional Amendments was widely believed to be enacted to overcome the judicial interpretations of the Fundamental Rights?
(a) 1st Amendment
(b) 42nd Amendment
(c) 44th Amendment
(d) 86th Amendment

Ans: A

Difficulty: Easy

Subject: Polity

Solution: 1st amendment added ninth schedule in constitution. Acts put in schedule 9 were saved from judicial review.

Controversial Question: 42nd amendment also tried to make the legislature strong vis-a-vis the judiciary. The question asks "widely believed"- makes it open for subjective interpretation.

Source: https://www.india.gov.in/my-government/constitution-india/amendments/constitution-india-first-amendment-act-1951



Q15. Consider the following organizations/ bodies in India:
1. The National Commission for Backward Classes
2.  The National Human Rights Commission
3. The National Law Commission
4. The National Consumer Disputes Redressal Commission
How many of the above constitutional bodies?
(a) Only one
(b) Only two
(c) Only three
(d) All four

Ans: A

Difficulty: Easy

Subject: Polity

Solution: From given options, Only the National Commission for Backward Classes is Constitutional Body (Art. 338B)



Q16. Consider the following statements:

  1. If the election of the President of India is declared void by the Supreme Court of India, all acts done by him/her in the performance of duties of his/her office of President before the date of decision become invalid.
  2. Election for the post of the President of India can be postponed on the grounds that some Legislative Assemblies have been dissolved and elections are yet to take place.
  3. When a Bill is presented to the President of India, the Constitution prescribes time limits within which he/she has to declare his/her assent.

How many of the above statements are correct?

(a) Only one
(b) Only two
(c) Al three
(d) None

Ans: D

Difficulty: Easy

Subject: Polity

Solution: All options are wrong.

Option 1 is incorrect: Article 71(2) of Constitution: If the election of a person as President or Vice President is declared void by the Supreme court, acts done by him in the exercise and performance of the powers and duties of the office of President or Vice President, as the case may be, on or before the date of the decision of the Supreme Court shall not be invalidated by reason of that declaration. (https://indiankanoon.org/doc/1349319/)

Option 2 is incorrect: Article 71(4) of Constitution: The election of a person as President or Vice President shall not be called in question on the ground of the existence of any vacancy for whatever reason among the members of the electoral college electing him (https://indiankanoon.org/doc/692848/)

Option 3 is incorrect: Article 111 of Constitution: Assent to Bills When a Bill has been passed by the Houses of Parliament, it shall be presented to the President, and the President shall declare either that he assents to the Bill, or that he withholds assent therefrom Provided that the President may, as soon as possible after the presentation to him. (https://indiankanoon.org/doc/692848/)



Q17. With reference to Finance Bill and Money Bill in the Indian Parliament,consider the following statements:

  1. When the Lok Sabha transmits the Finance Bill to the Rajya Sabha, it can amend or reject the Bill.
  2. When the Lok Sabha transmits Money Bill to the Rajya Sabha, it cannot amend or reject the Bill, it can only make recommendations.
  3. In the case of disagreement between the Lok Sabha and the Rajya Sabha, there is no joint sitting for Money Bill, but a joint sitting becomes necessary for the Finance Bill.

How many of the above statements are correct?
(a) Only one
(b) Only two
(c) All three
(d) None

Ans: A

Difficulty: Hard

Subject: Polity

Solution: 1 and 3 are not correct, 2 is correct.

Controversial Question: The report given by CAG (screenshot below) clearly states that Finance bill is a money bill under Article 110(1)(a). The problem lies in third statement where both Finance Bill and money bill are mentioned together, creating a room for doubt that Finance Bill here refers to Financial Bills under Article 117(1) and Article 117(3).

Compliance report of CAG

(https://cag.gov.in/webroot/uploads/download_audit_report/2016/Union_Civil_Compliance_Report_27_2016_Full.pdf)

A Finance Bill is a Money Bill as defined in Article 110 of the Constitution. Whereas a Financial Bill is an ordinary bill as it apart from dealing with money matters also deals with non-money matters.

  • It deals with the proposals of the government for levy of new taxes, modification of the existing tax structure or continuance of the existing tax structure beyond the period approved by Parliament are submitted to Parliament through this bill. So, it is introduced as a part of the Annual Financial Statement (i.e. Budget) under Article 112.
  • The Finance Bill is accompanied by a Memorandum containing explanations of the provisions included in it. The Finance Bill can be introduced only in Lok Sabha.
  • However, the Rajya Sabha can only recommend amendments in the Bill. The bill has to be passed by the Parliament within 75 days of its introduction. So, statement 1 is not correct and statement 2 is correct.
  • Statement 3 is not correct: As a finance bill is a money bill so no joint sitting of the two houses is allowed with regard to a finance bill under Article 108.

Source: M Laxmikanth



Q18. Consider the following statements:
Once the Central Government notifies an area as a 'Community Reserve'

  1. the Chief Wildlife Warden of the State becomes the governing authority of such forest
  2. hunting is not allowed in such area
  3. people of such area are allowed to collect non-timber forest produce
  4. people of such area are allowed traditional agriculture practices

How many of the above statements are correct?
(a) Only one
(b) Only two
(c) Only three
(d) All four

Ans: B

Difficulty: Medium

Subject: Environment

Solution: Statement 1 is not correct: Community Reserve Management Committee is the governing authority of such forest

Note: According to a recent amendment in the Wildlife Protection Act - the Chief Wildlife Warden of the State becomes the governing authority of Sanctuaries (sanctuaries and community reserves are different!)

Statement 2 is correct and 4 is not correct: After a forest has been made into a community reserve, people cannot hunt there, nor can they use it for agricultural practices, leave alone jhum cultivation.

Statement 3 is correct: people of such areas are allowed to collect non-timber forest produce.

Source: https://tribal.nic.in/downloads/FRA/Concerned%20Laws%20and%20Policies/Wildlife%20Protection%20Act,%201972.pdf



Q19. With reference to 'Scheduled Areas in India', consider the following statements:

  1. Within a State, the notification of an area as Scheduled Area takes place through an Order of the President.
  2. The largest administrative unit forming the Scheduled Area is the District and the lowest is the cluster of villages in the Block.
  3. The Chief Ministers of the concerned States are required to submit annual reports to the Union Home Ministry on the administration of Scheduled Areas in the States.

How many of the above statements are correct?
(a) Only one
(b) Only two
(c) All three
(d) None

Ans: B

Difficulty: Medium

Subject: Polity

Solution: 1 and 2 are correct.

Statement 1 is correct: As per Article 244(1) of the Constitution, expression Scheduled Areas means such areas as the President may by order declare to be Scheduled Areas.

(https://tribal.nic.in/downloads/CLM/CLM_Reports/6.pdf)

Statement 2 is correct

Statement 3 is not correct:  The Governor of the concerned States are required to submit annual reports to the President on the administration of Scheduled Areas in the States.



Q20. Consider the following statements:
Statement-I: The Supreme Court of India has held in some judgments that the reservation policies made under Article 16(4) of the Constitution of India would be limited by Article 335 for maintenance of efficiency of administration.
Statement-II: Article 335 of the Constitution of India defines the term 'efficiency of administration'.
Which one of the following is correct in respect of the above statements?
(a) Both Statement-I and Statement-II are correct and Statement-II is the correct explanation for Statement-I
(b) Both Statement-I and Statement-II are correct and Statement-II is not the correct explanation for Statement-I
(c) Statement-I is correct but Statement-II is incorrect
(d) Statement-I is incorrect but Statement-II is correct

Ans: C

Difficulty: Easy

Subject: Polity

Solution: Statement 1 is correct: Supreme Court in various judgements such as Indra Sawhney, M Nagraj etc. held that reservation policies made under Article 16(4) of the Constitution of India would be limited by Article 335 for maintenance of efficiency of administration.

Statement 2 is not correct: Constitution does not define the term “efficiency of administration” under Article 335.

Source: https://indiankanoon.org/doc/692848/



Q21. Consider the following statements
Statement-I: India, despite having uranium deposits, depends on coal for most of its electricity production.
Statement-II: Uranium, enriched to the extent of at least 60%, is required for the production of electricity.
Which one of the following is correct in respect of the above statements?
(a) Both Statement-I and Statement-II are correct and Statement-II is the correct explanation for Statement-I
(b) Both Statement-I and Statement-II are correct and Statement-II is not the correct explanation for Statement-1
(c) Statement-I is correct but Statement-II is incorrect
(d) Statement-I is incorrect but Statement-II is correct

Ans: C

Difficulty: Easy

Subject: Environment

Solution: Statement is correct: In India, around 49.3% electricity is generated using coal (https://powermin.gov.in/en/content/power-sector-glance-all-india)

Statement 2 is not correct: Uranium enriched to around 3-5% is required for production of electricity (not 60%).

(https://world-nuclear.org/information-library/nuclear-fuel-cycle/conversion-enrichment-and-fabrication/uranium-enrichment.aspx)



Q22. Consider the following statements :
Statement-I: Marsupials are not naturally found in India.
Statement-II: Marsupials can thrive only in montane grasslands with no predators.
Which one of the following is correct in respect of the above statements?
(a) Both Statement-I and Statement-II are correct and Statement-II is the correct explanation for Statement-I
(b) Both Statement-I and Statement-II are correct and Statement-II is not the correct explanation for Statement-I
(c) Statement-I is correct but Statement-II is incorrect
(d) Statement-1 is incorrect but Statement-II is correct

Ans: C

Difficulty: Easy

Subject: Environment

Solution: Statement 1 is correct:

(https://www.britannica.com/animal/marsupial)


Statement 2 is not correct: In South America, marsupials live in forests or tropical rainforests. (https://www.britannica.com/animal/marsupial)



Q23. Invasive Species Specialist Group' (that develops Global Invasive Species Database) belongs to which one of the following organizations?
(a) The International Union for Conservation of Nature
(b) The United Nations Environment Programme
(c) The United Nations World Commission for Environment and Development
(d) The World Wide Fund for Nature

Ans: A

Difficulty: Medium

Subject: Environment

Source: https://www.iucn.org/sites/default/files/2023-04/2021-iucn-ssc-invasive-species-sg-report_publication.pdf



Q24. Consider the following fauna:
1. Lion-tailed Macaque
2. Malabar Civet
3. Sambar Deer
How many of the above are generally nocturnal or most active after sunset?
(a) Only one
(b) Only two
(c) All three
(d) None

Ans: B

Difficulty: Hard

Subject: Environment

Solution: 2 and 3 are correct.

1. Lion-tailed Macaque is diurnal, meaning it is active exclusively in daylight hours.

2. https://www.conservationindia.org/articles/the-elusive-malabar-civet

3. https://animalia.bio/sambar?custom_list=571



Q25. Which of the following organisms perform waggle dance for others of their kin to indicate the direction and the distance to a source of their food?
(a) Butterflies
(b) Dragonflies
(c) Honeybees
(d) Wasps

Ans: C

Difficulty: Medium

Subject: Environment

Source: https://www.science.org/doi/10.1126/science.ade1702



Q26. Consider the following statements:

  1. Some mushrooms have medicinal properties.
  2. Some mushrooms have psychoactive properties.
  3. Some mushrooms have insecticidal properties.
  4. Some mushrooms have bioluminescent properties.

How many of the above statements are correct?
(a) Only one
(b) Only two
(c) Only three
(d) All four

Ans: D

Difficulty: Easy

Subject: Environment

Solution: Statement 1 is correct: Mushrooms act as antibacterial, immune system enhancer and cholesterol lowering agents; additionally, they are important sources of bioactive compounds. As a result of these properties, some mushroom extracts are used to promote human health and are found as dietary supplements.

(https://www.ncbi.nlm.nih.gov/pmc/articles/PMC4320875/#:~:text=Mushrooms%20act%20as%20antibacterial%2C%20immune,are%20found%20as%20dietary%20supplements.)

Statement 2 is correct: Species of the genera Conocybe, Gymnopilus, Panaeolus, Pluteus, Psilocybe, and Stropharia have psychoactive properties.

(https://pubmed.ncbi.nlm.nih.gov/16401965/#:~:text=These%20%22magic%22%2C%20psychoactive%20fungi,similar%20to%20lysergic%20acid%20diethylamide.)

Statement 3 is correct: Some mushrooms have insecticidal properties.

(https://www.sciencedirect.com/science/article/abs/pii/0031942295007733)

Statement 4 is correct: Certain mushrooms release a glow of cold light due to a chemical reaction between oxyluciferin molecules, an enzyme called luciferase, and oxygen

(https://www.forbes.com/sites/linhanhcat/2019/10/08/mushrooms-glow-in-the-dark/?sh=5d5a12234e70)



Q27. Consider the following statements regarding the Indian squirrels:

  1. They build nests by making burrows in the ground.
  2. They store their food materials like nuts and seeds in the ground.
  3. They are omnivorous.

How many of the above statements are correct?
(a) Only one
(b) Only two
(c) All three
(d) None

Ans: C

Difficulty: Medium

Subject: Environment

Solution:

Statement 1 is correct: Indian squirrels live in tropical dry forests, rainforests, mangrove forests, grasslands, scrub, parks, gardens, and urban areas. They live in a system of burrows that they use for shelter and storage.

Statement 2 is correct: They store their food materials like nuts and seeds in the ground. Squirrels hide nuts this way as preparation for cold weather.

Statement 3 is correct: Indian squirrels feed on nuts and fruits but will also eat seeds, insects, small mammals and reptiles, eggs, and sometimes chicks of birds.



Q28. Consider the following statements

  1. Some microorganisms can grow in environments with temperature above the boiling point of water.
  2. Some microorganisms can grow in environments with temperature below the freezing point of water.
  3. Some microorganisms can grow in a highly acidic environment with a pH below 3.

How many of the above statements are correct?
(a) Only one
(b) Only two
(c) All three
(d) None

Ans: C

Difficulty: Easy

Subject: Environment

Solution:

Statement 1 is correct: Thermophiles are microorganisms that can grow in high-temperature environments. Some thermophiles can grow in temperatures as high as 113°C (Boiling Point of water is 100°C). Eg. Pyrococcus furiosus

(https://theconversation.com/microbes-in-extreme-heat-and-cold-hold-lessons-about-life-on-earth-and-beyond-60208)

Statement 2 is correct: https://www.ncbi.nlm.nih.gov/pmc/articles/PMC92138/

Statement 3 is correct: Acidophiles are microorganisms that show optimal growth in highly acidic environments. These are of two types. The extreme acidophiles dwell in environments with a pH value <3, and moderate acidophiles grow optimally in conditions having pH values ranging between 3 and 5.

(https://www.sciencedirect.com/topics/immunology-and-microbiology/acidophile#:~:text=3%20Acidophiles,ranging%20between%203%20and%205.)



Q29. Which one of the following makes a tool with a stick to scrape insects from a hole in a tree or a log of wood?
(a) Fishing cat
(b) Orangutan
(c) Otter
(d) Sloth Bear

Ans: B

Difficulty: Medium

Subject: Environment

Solution: https://www.youtube.com/watch?v=DjXM5TR3hn4



Q30. Consider the following:
1. Aerosols
2. Foam agents
3. Fire retardants
4. Lubricants
In the making of how many of the above are hydrofluorocarbons used?
(a) Only one
(b) Only two
(c) Only three
(d) All four

Ans: D

Difficulty: Medium

Subject: Environment

Solution: Hydrofluorocarbons (HFCs) are greenhouse gases (GHGs) commonly used in a wide variety of applications, including refrigeration, air-conditioning (AC), building insulation, foam agents, and aerosols with minor uses as solvents and for fire protection.

(https://www.epa.gov/snap/reducing-hydrofluorocarbon-hfc-use-and-emissions-federal-sector-through-snap)

HFCs are also used in Lubricants (https://www.osti.gov/biblio/215711)



Q31. Consider the following statements:

  1. Jhelum River passes through Wular Lake.
  2. Krishna River directly feeds Kolleru Lake.
  3. Meandering of Gandak River formed Kanwar Lake.

How many of the statements given above are correct?
(a) Only one
(b) Only two
(c) All three
(d) None

Ans: B

Difficulty: Medium

Subject: Geography

Solution: Statement 1 is correct:

Statement 2 is not correct: Kolleru Lake is fed directly by water from the seasonal Budameru and Tammileru streams

Statement 3 is correct: Kanwar Lake (Kabar Taal) is the largest freshwater lake in Bihar.  It is a residual oxbow lake, formed due to meandering of Gandak river, a tributary of Ganga, in the geological past.

Note: Kanwar lake is located near Burhi Gandak River (Gandak and Burhi Gandak are different rivers).



Q32. Consider the following pairs:

  1. Kamarajar Port : First major port in India registered as a company
  2. Mundra Port : Largest privately owned port in India
  3. Vishakhapatnam Port: Largest container port in India

How many of the above pairs correctly matched?
(a) Only one pair
(b) Only two pairs
(c) All three pairs
(d) None of the pairs

Ans: B

Difficulty: Hard

Subject: Geography

Solution: 1, 2 are correct. JNPT (Jawaharlal Nehru Port Trust) is the largest container port in India)




Q33. Consider the following trees :

  1. Jackfruit (Artocarpus heterophyllus)
  2. Mahua (Madhuca indica)
  3. Teak (Tectona grands)

How many of the above are deciduous trees?
(a) Only one
(b) Only two
(c) All three
(d) None

Ans: B

Difficulty: Hard

Subject: Geography

Solution: 2 and 3 are correct.

  1. Jackfruit (Artocarpus heterophyllus) is an evergreen tree.
  2. Mahua (Madhuca indica): found in the dry deciduous type of forests of Chhattisgarh and Jharkhand.
  3. Teak (Tectona grandis) is a moist deciduous tree mainly found in North East India.


Q34. Consider the following statements

  1. India has more arable area than China.
  2. The proportion of irrigated area is more in India as compared to China.
  3. The average productivity per hectare in Indian agriculture is higher than that in China.

How many of the above statements are correct?
(a) Only one
(b) Only two
(c) All three
(d) None

Ans: B

Difficulty: Hard

Subject: Geography

Solution: Statement 1 is correct.

https://data.worldbank.org/indicator/AG.LND.ARBL.HA?locations=IN-CN

Statement 2 is correct:  China’s irrigation cover is 41% of cultivated area, and India’s is 48%.

(https://www.financialexpress.com/opinion/india-can-learn-agri-policy-lessons-from-china/1748398/)

Statement 3 is not correct: China’s productivity in most crops is 50 to 100% higher than India’s.

(https://prsindia.org/policy/analytical-reports/state-agriculture-india)



Q35. Which one of the following is the best example of repeated falls in sea level, giving rise to present-day extensive marshland?
(a) Bhitarkanika Mangroves
(b) Marakkanam Salt Pans
(c) Naupada Swamp
(d) Rann of Kutch

Ans: D

Difficulty: Medium

Subject: Geography

Solution: During 7000-3000 BC, the present area of the Rann was an arm of the Arabian Sea. The present low lying plains near the Rann or the Indus valley plains towards the sea had different phases - shallow sea to delta and then to the plains or Rann.

Source: https://forests.gujarat.gov.in/writereaddata/images/pdf/26_Rann-Booklet.pdf



Q36. Ilmenite and rutile, abundantly available in certain coastal tracts of India, are rich sources of which one of the following?
(a) Aluminium
(b) Copper
(c) Iron
(d) Titanium

Ans: D

Difficulty: Hard

Subject: Geography

Solution: Ilmenite (FeO.TiO2) and rutile (TiO2) are the two chief minerals of titanium

Source: https://ibm.gov.in/writereaddata/files/08172015131610Ilmenite%20and%20Rutile.pdf



Q37. About three-fourths of world's cobalt, a metal required for the manufacture of batteries for electric motor vehicles, is produced by
(a) Argentina
(b) Botswana
(c) the Democratic Republic of the Congo
(d) Kazakhstan

Ans: C

Difficulty: Medium

Subject: Geography

Solution: The leading country in worldwide cobalt mine production in 2022 was the Democratic Republic of Congo, having produced an estimated 130,000 metric tons that year.

Source: https://www.statista.com/statistics/264928/cobalt-mine-production-by-country/#:~:text=The%20leading%20country%20in%20worldwide,130%2C000%20metric%20tons%20that%20year.



Q38. Which one of the following is a part of the Congo Basin?

(a) Cameroon
(b) Nigeria
(c) South Sudan
(d) Uganda

Ans: A

Difficulty: Easy

Subject: Geography

Solution:

Map of Congo Basin:




Q39. Consider the following statements:

  1. Amarkantak Hills are at the confluence of Vindhya and Sahyadri Ranges.
  2. Biligirirangan Hills constitute the easternmost part of Satpura Range.
  3. Seshachalam Hills constitute the southernmost part of Western Ghats.

How many of the statements given above are correct?
(a) Only one
(b) Only two
(c) All three
(d) None

Ans: D

Difficulty: Medium

Subject: Geography

Solution: Statement 1 is not correct:Amarkantak Hills are located in Madhya Pradesh, near Chhattisgarh border. Whereas Sahyadri is in Western Ghats (Maharashtra).

Statement 2 is not correct: The Biligirirangan Hills are located in the Chamarajanagar district of Karnataka, India. They are part of the Eastern Ghats mountain range. Saputara range passes through Maharashtra and Madhya Pradesh states.

Statement 3 is not correct: The Seshachalam Hills, also known as the Tirumala Range are hill ranges of the Eastern Ghats (not western).

Source: https://www.mapsofindia.com/maps/india/physical-map.html



Q40. With reference to India's projects on connectivity, consider the following statements:

  1. East-West Corridor under Golden Quadrilateral Project connects Dibrugarh and Surat.
  2. Trilateral Highway connects Moreh in Manipur and Chiang Mai in Thailand via Myanmar.
  3. Bangladesh-China-India-Myanmar Economic Corridor connects Varanasi in Uttar Pradesh with Kunming in China.

How many of the above statements are correct?
(a) Only one
(b) Only two
(c) All three
(d) None

Ans: D

Difficulty: Medium

Subject: Geography

Solution: Statement 1 is not correct: The Golden Quadrilateral Project connects the four major metropolitan cities: Delhi, Mumbai, Chennai, and Kolkata.

The East-West Corridor is a part of this project and connects Silchar in Assam to Porbandar in Gujarat.

Statement 2 is not correct: The IMT Trilateral Highway connects Moreh in the Indian state of Manipur with Mae Sot in Thailand, passing through Myanmar.

Statement 3 is not correct: Bangladesh-China-India-Myanmar Economic Corridor connects Kolkata in Uttar Pradesh with Kunming in China




Q41. Consider the following statements:
Statement-I: Interest income from the deposits in Infrastructure Investment Trusts (InvITs) distributed to their investors is exempted from tax, but the dividend is taxable.
Statement-II: InvITs are recognized as borrowers under the 'Securitization and Reconstruction of Financial Assets and Enforcement of Security Interest Act, 2002.
Which one of the following is correct in respect of the above statements?
(a) Both Statement-I and Statement-II are correct and Statement-II is the correct explanation for Statement-I
(b) Both Statement-I and Statement-II are correct and Statement-II is not the correct explanation for Statement-I
(c) Statement-I is correct but Statement-II is incorrect
(d) Statement-I incorrect but Statement-ll is correct

Ans: D

Difficulty: Medium

Subject: Economy

Solution: Under the latest budget 2023-24, all income distributed by InvITs, including interest income, dividend income, and rental income, will be taxable in the hands of unitholders. This is a change from the previous tax regime, under which only dividend income from InvITs was taxable.

Some additional details about the tax treatment of InvITs under the latest budget:

  • Interest income from deposits in InvITs will be taxed at the investor’s marginal income tax rate.
  • Dividend income from InvITs will be taxed at the dividend distribution tax (DDT) rate of 15%.
  • Rental income from InvITs will be taxed at the investor’s marginal income tax rate

Statement 2 is correct: InvITs are recognized as borrowers under the 'Securitization and Reconstruction of Financial Assets and Enforcement of Security Interest Act, 2002.

Source:

  1. https://www.thehindubusinessline.com/news/real-estate/reits-invits-to-approach-fin-min-against-tax-on-distribution-income/article66481515.ece
  2. https://www.lexology.com/commentary/banking-financial-services/india/azb-partners/fpi-investment-in-debt-instruments-by-reits-and-invits


Q42. Consider the following statements:
Statement-I: In the post-pandemic recent past, many Central Banks worldwide had carried out interest rate hikes.
Statement-II: Central Banks generally assume that they have the ability to counteract the rising consumer prices via monetary policy means.
Which one of the following is correct in respect of the above statements?
(a) Both Statement-I and Statement-II are correct and Statement-II is the correct explanation for Statement-I
(b) Both Statement-I and Statement-II are correct and Statement-II is not the correct explanation for Statement-I
(c) Statement-I is correct but Statement-II is incorrect
(d) Statement-I is correct incorrect but Statement-II is correct

Ans: A

Difficulty: Easy

Subject: Economy

Solution:

Source: Economic Survey



Q43. Consider the following statements:
Statement-I: Carbon markets are likely to be one of the most widespread tools in the fight against climate change.
Statement-II: Carbon markets transfer resources from the private sector to the State.
Which one of the following is correct in respect of the above statements?
(a) Both Statement-| and Statement-II are correct and Statement-II is the correct explanation for Statement-I
(b) Both Statement-I and Statement-II are correct and Statement-II is not the correct explanation for Statement-I
(c) Statement-I is correct but Statement-II is incorrect
(d) Statement-I is incorrect but Statement-II is correct

Ans: B/C

Difficulty: Hard

Subject: Environment

Solution: Statement 1 is correct: Carbon markets are mechanisms created to fight against climate change by reducing the use of fossil fuels.

(https://climatepromise.undp.org/news-and-stories/what-are-carbon-markets-and-why-are-they-important)

Statement 2 is correct: Going by the Economist Article (https://www.economist.com/finance-and-economics/2022/05/26/carbon-markets-are-going-global) statement 2 is correct.

Controversial Question: The answer depends on how you define the carbon market. If only trading platforms are considered, then option 2 will be wrong. If one considers other mechanisms of carbon pricing such as carbon tax, then statement 2 becomes correct.

(https://carbonpricingdashboard.worldbank.org/what-carbon-pricing)



Q44. Which one of the following activities of the Reserve Bank of India is considered to be part of 'sterilization?
(a) Conducting 'Open Market Operations'
(b) Oversight of settlement and payment systems
(c) Debt and cash management for the Central and State Governments
(d) Regulating the functions of Non-banking Financial Institutions

Ans: A

Difficulty: Easy

Subject: Economy

Solution: Sterilization is a form of monetary action in which a central bank seeks to limit the effect of inflows and outflows of capital on the money supply. Sterilization most frequently involves the purchase or sale of financial assets by a central bank and is designed to offset the effect of foreign exchange intervention. Example of sterilization is 'Open Market Operations' .

Source: https://www.investopedia.com/terms/s/sterilization.asp



Q45. Consider the following markets
1. Government Bond Market
2. Call Money Market
3. Treasury Bill Market
4. Stock Market
How many of the above are included in capital markets?
(a) Only one
(b) Only two
(c) Only three
(d) All four

Ans: B

Difficulty: Easy

Subject: Economy

Solution: 1 and 4 are capital markets. 2, 3 are money markets.

Source: NCERT



Q46. Which one of the following best describes the concept of 'Small Farmer Large Field'?
(a) Resettlement of a large number of people, uprooted from their countries due to war, by giving them a large cultivable land which they cultivate collectively and share the produce
(b) Many marginal farmers in an area organize themselves into groups and synchronize and harmonize selected agricultural operations
(c) Many marginal farmers in an area together make a contract with a corporate body and surrender. their land to the corporate body for a fixed term for which the corporate body makes a payment of agreed amount to the farmers
(d) A company extends loans, technical knowledge and material inputs to a number of small farmers in an area so that they produce the agricultural commodity required by the company for its manufacturing process and commercial production

Ans: B

Difficulty: Medium

Subject: Economy

Solution: The concept of “Small Farmer Large Field” refers to the idea where many marginal farmers in a particular area come together and organize themselves into groups. These groups work collectively to synchronize and harmonize selected agricultural operations.

In this approach, instead of individually working on their small plots of land, these farmers pool their resources, knowledge, and efforts to cultivate larger fields collectively. By doing so, they can achieve economies of scale, improve efficiency, and enhance productivity.

This concept allows small farmers to overcome the limitations of their individual small land holdings and collaborate to optimize their agricultural activities. It can involve sharing equipment, sharing labor, collectively marketing their produce, and adopting modern farming techniques.


Source: https://link.springer.com/article/10.1007/s12571-021-01236-x



Q47. Consider the following statements:

  1. The Government of India provides Minimum Support Price for niger (Guizotia abyssinica) seeds.
  2. Niger is cultivated as a Kharif crop.
  3. Some tribal people in India use niger seed oil for cooking.

How many of the above statements are correct?
(a) Only one
(b) Only two
(c) All three
(d) None

Ans: C

Difficulty: Easy (as this crop is under MSP)

Subject: Economy

Solution: Statement 1 and 2 are correct: One of the 14 kharif crops for which the Centre releases a minimum support price (MSP) every year is an unlikely plant called niger or ramtil (Guizotia abyssinica). This lesser known oilseed commands one of the highest MSPs, fixed on the basis of the cost of production and market demand.

Statement 3 is correct: The tribal population uses niger seed oil for cooking, the press cake post oil-extraction as livestock feed, and also consume the seeds as a condiment. Niger seed oil has medicinal properties, which is the reason for its commercial demand by the cosmetics, perfumeries and other allied industries.

Source: https://www.downtoearth.org.in/news/agriculture/illusive-oilseed-india-s-niger-seed-cultivation-is-declining-here-is-why-84380



Q48. Consider the investments in the following assets:

  1. Brand recognition
  2. Inventory
  3. Intellectual property
  4. Mailing list of clients

How many of the above are considered intangible investments?
(a) Only one
(b) Only two
(c) Only three
(d) All four

Ans: C

Difficulty: Medium

Subject: Economy

Solution: Intangible assets or intangible investments are those which cannot be touched physically (non-physical assets). Its examples are Brand Recognition, Intellectual Property, Mailing list, etc.

Tangible assets are those which can be touched (physical assets) - eg. inventory, land etc.

Source: https://www.investopedia.com/terms/i/intangibleasset.asp



Q49. Consider the following:

  1. Demographic performance
  2. Forest and ecology
  3. Governance reforms
  4. Stable government
  5. Tax and fiscal efforts

For the horizontal tax devolution, the Fifteenth Finance Commission used how many of the above as criteria other than population area and income distance?
(a) Only two
(b) Only three
(c) Only four
(d) All five

Ans: B

Difficulty: Easy

Subject: Economy

Solution: 1, 2 and 5 are correct.

Source: https://prsindia.org/policy/report-summaries/report-15th-finance-commission-2021-26



Q50. Consider the following infrastructure sectors:

  1. Affordable housing
  2. Mass rapid transport
  3. Health care
  4. Renewable energy

How many of the above does the UNOPS Sustainable Investments in Infrastructure and Innovation (S3i) initiative focus for its investments?
(a) Only one
(b) Only two
(c) Only three
(d) All four

Ans: C

Difficulty: Hard

Subject: Economy

Solution: 1, 3 and 4 are correct: S3i Infrastructure's core business sectors include: renewable energy, health care, and affordable housing. We invest in emerging markets. Our renewable energy projects include solar, hydro and wind energy

Source: https://content.unops.org/documents/S3I-Renewable-Energy-Portfolio.pdf



Q51.With reference to home guards, consider the following statements:

  1. Home Guards are raised under the Home Guards Act and Rules of the Central Government.
  2. The role of the Home Guards is to serve as an auxiliary force to the police in maintenance of internal security
  3. To prevent infiltration on the international border / coastal areas, the Border Wing Home Guards Battalions have been raised in some States.

How many of the above statements are correct?
(a) Only one
(b) Only two
(c) All three
(d) None

Ans: B

Difficulty: Medium

Subject: Polity

Solution: 1 is wrong. 2, 3 are right.

1 is wrong: Home Guards are raised under the Home Guards Act and Rules of the States/Union Territories.

2 is correct: They are deployed as auxillary force in maintaining Law and Order with Police.

(https://home.rajasthan.gov.in/content/homeportal/en/homeguardsdepartment/homeguards/borderwinghomeguard.html)

3 is correct: Role of the Border Wing Home Gaurd (B.H.G.) is to work in conjunction with the army in guarding the vital Installations, the vital points and the vulnerable areas during break out of hostilities on the International Border.

(https://home.rajasthan.gov.in/content/homeportal/en/homeguardsdepartment/homeguards/borderwinghomeguard.html)



Q52. With reference to India, consider the following pairs:

Sr. No.

Action

The Act under which it is covered

1.

Unauthorized wearing of police or military uniforms

The Official Secrets Act, 1923

2.

Knowingly misleading or otherwise interfering with a police officer or military officer when engaged in their duties

The Indian Evidence Act, 1872

3.

Celebratory gunfire which can endanger the personal safety of others

The Arms (Amendment) Act, 2019

How many of the above pairs are correctly matched?

a) Only one

b) Only two

c) All three

d) None

Ans: B

Difficulty: Hard

Subject: Polity

Solution: 1 and 3 are correct.

Statement 1 is correct:

(https://www.indiacode.nic.in/bitstream/123456789/2379/1/A1923-19.pdf)

Statement 2 is not correct: Official secrets act stated that No person in the vicinity of any prohibited place shall obstruct, knowingly mislead or otherwise interfere with or impede, any police officer, or any member of engaged on guard, Sentry, patrol, or other similar duty in relation to the prohibited place.


Statement 3 is correct:

(https://www.mha.gov.in/sites/default/files/ActAndRuleThe%20ArmsAct_17122019.pdf)




Q53. Consider the following pairs:

Sr No. 

Regions often mentioned in news

Reason for being in news

1.

North Kivu and Ituri

War between Armenia and Azerbaijan

2.

Nagorno-Karabakh

Insurgency in Mozambique

3.

Kherson and zaporizhzhya

Dispute between Israel and Lebanon

How many of the above pairs are correctly matched?

a) Only one

b) Only two

c) All three

d) None

Ans: D

Difficulty: Medium

Subject: Current Affairs

Solution:

Correct Pairs are:

1. North Kivu - Congo region

2. Nagorno - Azerbaijan

3. Kherson and Zaporizhzhia - Ukrainian territory controlled by Russia



Q54. Consider the following statements

Statement-1: Israel has established diplomatic relations with some Arab States.

Statement-II: The 'Arab Peace Initiative' mediated by Saudi Arabia was signed by Israel and Arab League.
Which one of the following is correct in respect of the above statements?

(a) Both Statement-I and Statement-II are correct and Statement-II is the correct explanation for Statement-I

(b) Both Statement-I and Statement-II are correct and Statement-II is not the correct explanation for Statement-I

(c) Statement-I is correct but Statement-II is incorrect

(d) Statement-I is incorrect but Statement-II is correct

Ans: C

Difficulty: Hard

Subject: Current Affairs

Solution: Israel has established diplomatic relations with some Arab States

Statement 1 is correct:

(https://foreignpolicy.com/2020/12/21/arab-ties-israel-diplomacy-normalization-middle-east/)

Statement 2 is not correct: Arab Peace Initiative was not signed by Israel. The Arab Peace Initiative, also known as the “Saudi Initiative,” was proposed by Saudi Arabia in 2002 at the Arab League Summit in Beirut. It was endorsed by the Arab League as a potential resolution to the Israeli-Palestinian conflict.

(https://www.telegraph.co.uk/news/worldnews/1547036/Israel-rejects-Arab-peace-initiative.html)



Q55.  Consider the following pairs with regard to sports awards.

  1.  

Major Dhyan Chand Khel Ratna Award

For the most spectacular and outstanding performance by a sportsperson over period of last four years

  1.  

Arjuna Award

For the lifetime achievement by a sportperson

3.

Dronocharya Award

To honour eminent coaches who have successfully trained sports person or teams

4.

Rashtriya Khel Protsahan Puraskar

To recognize the contribution made by sports persons even after their retirement

How many of the above pairs are correctly matched?

a) Only one

b) Only two

c) All three

d) All four

Ans: B

Difficulty: Medium

Subject: Current Affairs

Solution:

Pair 1 is correct: Major Dhyan Chand Khel Ratna Award:  For the "spectacular and most outstanding performance in the field of sports over a period of four years" at international level.

Pair 2 is not correct: Arjuna Awards for Outstanding Performance in Sports and Games, is the second-highest sporting honour of India. Dhyan chand award is the lifetime achievement sporting honour of India.

Pair 3 is correct: Dronacharya Award is given to coaches. It is given for doing outstanding and meritorious work on a consistent basis and enabling sportspersons to excel in International events.

Source: https://olympics.com/en/news/national-sports-awards-india



Q56. Consider the following statements in respect of the 44th Chess Olympiad, 2022 :

  1. It was the first time that the Chess Olympiad was held in India.
  2. The official mascot was named ‘Thambi'.
  3. The trophy for the winning team in the open section is the Vera Menchik Cup.
  4. The trophy for the winning team in the women's section is the Hamilton-Russell Cup.

How many of the statements given above are correct?
(a) Only one
(b) Only two
(c) Only three
(d) All four

Ans: B

Difficulty: Hard

Subject: Current Affairs

Solution:

Statement 1 and 2 are correct: For the 1st time ever, the world's biggest chess event is happening in India. Chennai is hosting the 44th Chess Olympiad.

The Official Mascot of 44th Chess Olympiad is 'Thambi'. The word 'Thambi' in Tamil language means - little or younger brother. So, statement 2 is correct.

Statement 3 and 4 are not correct (interchanged): 1st place in the Open section: Hamilton-Russel Cup.

1st place in the Women's section: Vera Menchik Cup. So, statement 4 is not correct.

Source: https://www.chess.com/events/2022-fide-chess-olympiad



Q57. Consider the following pairs:

Sr No. 

Area of conflict mentioned in news

Country where it is located

1.

Donbas

Syria

2.

Kachin

Ethiopia

3.

Tigray

North Yemen

How many of the above pairs are correctly matched?

a) Only one

b) Only two

c) All three

d) None

Ans: D

Difficulty: Medium

Subject: Current Affairs

Solution: 1 is not correct: The Donbas or Donbass is a historical, cultural, and economic region in eastern Ukraine.


2 is not correct: Kachin State is the northernmost state of Myanmar.


3 is not correct: Tigray is Ethiopia's northernmost region.



Q58. In recent years Chad, Guinea, Mali and Sudan caught the international attention for which one of the following reasons is common to all of them?
(a) Discovery of rich deposits of rare earth elements
(b) Establishment of Chinese military bases
(c) Southward expansion of Sahara Desert
(d) Successful coups

Ans: D

Difficulty: Hard

Subject: Current Affairs

Solution:

  1. https://www.washingtonpost.com/politics/2021/11/22/are-coups-really-contagious/
  2. https://www.theafricareport.com/153400/chad-guinea-mali-sudan-can-a-coup-be-a-springboard-for-democracy/


Q59. Consider the following heavy industries
1. Fertilizer plants
2. Oil refineries
3. Steel plants
Green hydrogen is expected to play a significant role in decarbonizing how many of the above industries?
(a) Only one
(b) Only two
(c) All three
(d) None

Ans: C

Difficulty: Easy

Subject: Economy

Solution:

(https://www.indiabudget.gov.in/economicsurvey/doc/echapter.pdf Page 227-28)

Source: Economic Survey 2022-23



Q60. Consider the following statements about G-20:

  1. The G-20 group was originally established as a platform for the Finance Ministers and Central Bank Governors to discuss international economic and financial issues.
  2. Digital public infrastructure is one of India's G-20 priorities.

Which of the statements given above is /are correct?
(a) 1 only
(b) 2 only
(c) Both 1 and 2
(d) Neither 1 nor 2

Ans: C

Difficulty: Easy

Subject: Current Affairs

Solution:

Statement 1 is correct: The G20 was founded in 1999 after the Asian financial crisis as a forum for the Finance Ministers and Central Bank Governors to discuss global economic and financial issues. (https://www.g20.org/en/about-g20/)

Statement 2 is correct: India is pitching its digital infrastructure at G20 as an open-access platform. Digital Public Infrastructure is one of the key priorities during India’s G20 Presidency. It is a cross-cutting theme across various Working Groups including Digital Economy, Health, Education and Global Partnership for Financial inclusion.

(https://government.economictimes.indiatimes.com/news/technology/indias-digital-public-infrastructure-can-help-developing-world-g20-sherpa-amitabh-kant/99134581)



Q61. With reference to the Indian History, Alexander Rea, A. H. Longhurst, Robert Sewell, James Burgess and Walter Elliot were associated with
(a) archaeological excavations
(b) establishment of English Press in Colonial India
(c) establishment of Churches in Princely States
(d) construction of railways in Colonial India

Ans: A

Difficulty: Hard

Subject: Modern Indian History

Solution: https://epgp.inflibnet.ac.in/epgpdata/uploads/epgp_content/S000829IC/P001688/M024813/ET/1510306093P07-M04-TracingthehistoryofIndianArchaeologyanditsPioneers-ET.pdf



Q62. Consider the following pairs:
Site          Well known for
1. Besnagar     : Shaivite cave shrine
2. Bhaja     : Buddhist cave shrine
3. Sittanavasal  : Jain cave shrine
How many of the above pairs are correctly matched?
(a) Only one
(b) Only two
(c) All three
(d) None

Ans: C

Difficulty: Hard

Subject: Ancient/Medieval/Art and Culture

Solution:

Pair 1 is correct: Cave 4 Known as Bina Cave in Vidhisha (Benagar), a stage in process can be seen in cave number four. An impressive Shiva Lingam is there inside Cave 4 over a rock-cut platform and one can access the cave through some delicate floral scrolls. Square moulded rocks are places near the door frames, and the cave is surrounded by two rock carved pillars and two 'dwara-palas' or door guardians.

(http://www.cpreecenvis.nic.in/Database/UdayagiriCave_2906.aspx?format=Print)

Pair 2 is correct: Bhaja Caves are the best example of Buddhist cave architecture in India. They are located on a hill village Bhaja in Pune district near Lonavala, Mumbai.

(https://www.maharashtratourism.gov.in/-/bhaja-caves)

Pair 3 is correct:

https://www.thehindu.com/news/national/tamil-nadu/sittanavasals-jain-monuments-weathering-the-elements-and-public-apathy/article65931192.ece



Q63. Consider the following statements:

Statement-I: 7th August is declared as the National Handloom Day.

Statement-II: It was in 1905 that the Swadeshi Movement was launched on the same day.

Which one of the following is correct with respect to the above statements ?

(a) Both Statement-I and Statement-II are correct and Statement-ll is the correct explanation for Statement

(b) Both Statement-I and Statement-are correct and Statement-II is not the correct explanation for Statement-I

(c) Statement-I is correct but Statement-Il is incorrect

(d) Statement-I is incorrect but Statement-II is correct

Ans: A

Difficulty: Hard

Subject: Modern Indian History

Solution: Statement 1 is correct: In 2015, the Government of India decided to designate the 7th August every year, as the National Handloom Day. The first National Handloom Day was inaugurated on 7 August 2015 by Prime Minister Narendra Modi in Chennai. On this day, we honour our handloom-weaving community and highlight the contribution of this sector in the socio-economic development of our country.

Statement 2 is correct: The Swadeshi Movement which was launched on 7th August, 1905 had encouraged indigenous industries and in particular handloom weavers. In 2015, the Government of India decided to designate the 7th August every year, as the National Handloom Day.

Source: https://www.mygov.in/campaigns/national-handloom-day/



Q64. Consider the following statements statements in respect of the National Flag of India according to the Flag Code of India, 2002
Statement-I: One of the the National standard Flag of sizes India of is 600 mm × 400 mm.
Statement-II: The ratio of the length to the height (width) of the Flag shall be 3: 2.
Which one of the following is correct in respect of the above statements?
(a) Both Statement-I and Statement-II are correct and Statement-II is the correct explanation for Statement- I
(b) Both Statement-I and Statement-II are correct and Statement-II is not the correct explanation for Statement-I
(c) Statement-I is correct but Statement-II is incorrect
(d) Statement-I is incorrect but Statement-II is correct

Ans: D

Difficulty: Hard

Subject: Current Affairs (Har Ghar Tiranga)

Solution:

Statement I is incorrect:

(https://www.mha.gov.in/sites/default/files/flagcodeofindia_070214.pdf)

Statement 2 is correct: The National Flag shall be rectangular in shape. The ratio of the length to the height (width) of the Flag shall be 3:2.



Q65. Consider the following statements in respect of the Constitution Day :
Statement-I: The Constitution Day is celebrated on 26th November every year to promote constitutional values among citizens.

Statement-II: On 26th November, 1949, the Constituent Assembly of India set up a Drafting Committee under the Chairmanship of Dr. B. R. Ambedkar to prepare a Draft Constitution of India.
Which one of the following is correct in respect of the above statements?
(a) Both Statement-I and Statement-II are correct and Statement-II is the correct explanation for Statement-I
(b) Both Statement-I and Statement-II are correct and Statement-II is not the correct explanation for Statement-I

(c) Statement-I is correct but Statement-II is incorrect
(d) Statement-I is incorrect  but Statement-II is correct

Ans: C

Difficulty: Easy

Subject: Polity

Solution:

Statement-I is correct: Constitution Day also known as 'Samvidhan Divas', is celebrated in our country on 26th November every year to commemorate the adoption of the Constitution of India. On 26th November 1949, the Constituent Assembly of India adopted the Constitution of India, which came into effect from 26th January 1950. The Ministry of Social Justice and Empowerment on 19th November 2015 notified the decision of Government of India to celebrate the 26th day of November every year as ‘Constitution Day’ to promote Constitution values among citizens.

(https://www.mygov.in/campaigns/constitution-day/)

Statement II is not correct: On 26th November 1949, the Constituent Assembly of India adopted the Constitution of India, which came into effect from 26th January 1950



Q66. Consider the following statements
Statement-I: Switzerland is one of the leading exporters of gold in terms of value.
Statement-II: Switzerland has the second largest gold reserves in the world.
Which one of the following is correct in respect of the above statements?
(a) Both Statement-I and Statement-II are correct and Statement-II is the correct explanation for Statement- I
(b) Both Statement-I and Statement-II are correct and Statement-II is not the correct explanation for Statement-I

(c) Statement-I is correct but Statement-II is incorrect
(d) Statement-I is incorrect but Statement-II is correct

Ans: C

Difficulty: Medium

Subject: Economy

Solution:

Statement I is correct: In 2021, Switzerland exported $86.7B in Gold, making it the 1st largest exporter of Gold in the world.

(https://oec.world/en/profile/bilateral-product/gold/reporter/che#:~:text=it%20the%201st-,largest,-exporter%20of%20Gold)

Statement II is incorrect: Switzerland has the seventh-largest reserves of gold in the world. Its reserves of 1,040.0 tons account for 6.3% of its foreign reserves.

Largest reserves of gold:

(https://www.investopedia.com/ask/answers/040715/what-countries-have-largest-gold-reserves.asp)



Q67. Consider the following statements

Statement-I: Recently, the United States of America (USA) and the European Union (EU) have launched the 'Trade and Technology Council'.

Statement-II: The USA and the EU claim that through this they are trying to bring technological progress and physical productivity under their control.

Which one of the following is correct in respect of the above statements?

(a) Both Statement-I and Statement-II are correct and Statement-II is the correct explanation for Statement-I

(b) Both Statement-I and Statement-II are correct and Statement-II is not the correct Statement-I explanation for Statement-I

(c) Statement-I is correct but Statement-II is incorrect

(d) Statement-I is incorrect but Statement-II is correct

Ans: C

Difficulty: Medium

Subject: Current Affairs

Solution:

Statement I is correct: The EU-US Trade and Technology Council serves as a forum for the United States and European Union to coordinate approaches to key global trade, economic, and technology issues and to deepen transatlantic trade and economic relations based on these shared values. It was established during the EU-US Summit on 15 June 2021 in Brussels.

(https://commission.europa.eu/strategy-and-policy/priorities-2019-2024/stronger-europe-world/eu-us-trade-and-technology-council_en)


Statement II is correct: The EU-US Trade and Technology Council serves as a forum for the United States and European Union to coordinate approaches to key global trade, economic, and technology issues and to deepen transatlantic trade and economic relations based on these shared values.


Q68. Consider the following statements :
Statement-I: India accounts for 3.2% of global export of goods.

Statement-II: Many local companies and some foreign companies operating in India have taken advantage of India's 'Production-linked Incentive' scheme.

Which one of the following is correct in respect of the above statements?

(a) Both Statement-I and Statement-II are correct and Statement-II is the correct explanation for Statement-I

(b) Both Statement-I and Statement-II are correct and Statement-II is not the correct explanation for Statement-I

(c) Statement-I is correct but Statement-II is incorrect

(d) Statement-I is incorrect but Statement-II is correct

Ans: D

Subject: Economy

Difficulty: Easy

Solution:

Statement I is not correct: India accounts for 1.8% of global export of goods. (Economic Survey)

Statement II is correct:

(Economic Survey 2022-23)



Q69. Consider the following statements:
The 'Stability and Growth Pact’ of the European Union is a treaty that

  1. limits the levels of the budgetary deficit of the countries of the European Union
  2. makes the countries of the European Union to share their infrastructure facilities
  3. enables the countries of European Union to share the their technologies

How many of the above statements are correct?
(a) Only one
(b) Only two
(c) All three
(d) None

Ans: A

Difficulty: Hard

Subject: Current Affairs

Solution: Statement 1 is correct, 2 and 3 are incorrect.

The Stability and Growth Pact (SGP) is a set of rules designed to ensure that countries in the European Union pursue sound public finances and coordinate their fiscal policies.

(https://economy-finance.ec.europa.eu/economic-and-fiscal-governance/stability-and-growth-pact_en#:~:text=The%20Stability%20and%20Growth%20Pact,and%20coordinate%20their%20fiscal%20policies.)


Q70. Consider the following statements:

  1. Recently, all the countries of the United Nations have adopted the first-ever compact for international migration, the 'Global Compact for Safe, Orderly and Regular Migration (GCM)'.
  2. The objectives and commitments stated in the GCM are binding on the UN member countries.
  3. The GCM addresses internal migration or internally displaced people also in its objectives and commitments.

How many of the above statements are correct?

(a) Only one
(b) Only two
(c) All three
(d) None

Ans: D

Difficulty: Hard

Subject: Current affairs

Solution:

Statement 1 is not correct: Out of the 193-member states, 164 countries have adopted the compact.

(https://www.epw.in/journal/2019/11/commentary/india-and-global-compact-migration.html)

Statement 2 is not correct: The Global Compact for Migration is the first-ever UN global agreement on a common approach to international migration in all its dimensions. The global compact is non-legally binding.

Statement 3 is not correct: Nothing as such is mentioned in objectives

(https://documents-dds-ny.un.org/doc/UNDOC/GEN/N18/451/99/PDF/N1845199.pdf?OpenElement)



Q71. Consider the following countries

  1. Bulgaria
  2. Czech Republic
  3. Hungary
  4. Latvia
  5. Lithuania
  6. Romania

How many of the above-mentioned countries share a land border with Ukraine?
(a) Only two
(b) Only three
(c) Only four
(d) Only five

Ans: A

Difficulty: Easy

Subject: Geography

Solution: 3 and 6 correct.




Q72. With reference to the Earth's atmosphere, which one of the following statements is correct?
(a) The total amount of insolation received at the equator is roughly about 10 times of that received at the poles.

(b) Infrared rays constitute roughly two-thirds of insolation.

(c) Infrared waves are largely absorbed that is by water vapour concentrated in the lower atmosphere.

(d) Infrared waves are a part of visible spectrum of electromagnetic waves of solar radiation.

Ans: C

Difficulty: Medium

Subject: Geography

Solution: Within the troposphere water vapour, ozone and other gases absorb much of the near infrared radiation.

Source: https://agupubs.onlinelibrary.wiley.com/doi/full/10.1029/2005JD006796



Q73. Consider the following statements:
Statement-I: The soil in tropical rain forests is rich in nutrients.

Statement-II: The high temperature and moisture of tropical rain forests cause dead organic matter in the soil to decompose quickly.

Which one of the following is correct in respect of the above statements?
(a) Both Statement-1 and Statement-II are correct and Statement-II is the correct explanation for Statement-I
(b) Both Statement-I and Statement-II are correct and Statement-II is not the correct explanation for Statement-I

(c) Statement-I is correct but Statement-II is incorrect
(d) Statement-I is incorrect but Statement-II is correct

Ans: D

Difficulty: Easy

Subject: Geography

Solution:

Statement I is not correct: What is interesting about the amount of precipitation in rainforests is that because they are so moist, the soil is typically less full of nutrients because they are being flushed out by constant rain. Therefore, many plants have adapted to reach and store nutrients off the ground.

Soils in tropical rainforests are typically deep but not very fertile, partly because large proportions of some mineral nutrients are bound up at any one time within the vegetation itself rather than free in the soil.

Source: https://www.britannica.com/science/tropical-rainforest



Q74. Consider the following statements:

Statement-I: The temperature contrast between continents and oceans is greater during summer than in winter.

Statement-II: The specific heat of water is more than that of land surface.

Which one of the following is correct in respect of the above statements?

(a) Both Statement-I and Statement-II are correct and Statement-II is the correct explanation for Statement-II

(b) Both Statement-I and Statement-II are correct and Statement-II is not the correct explanation for Statement-I

(c) Statement-I is correct but Statement-II is incorrect

(d) Statement-I is incorrect but Statement-II is correct

Ans: A

Difficulty: Hard

Subject: Geography

Solution:

Statement 1 is correct: The temperature contrast between continents and oceans is greater during summer than in winter. This is the result of more heat and stronger sea breeze in the summer than in winter.

Statement 2 is correct: The specific heat capacity of water is much greater than the land because the relative density of water is much lower than that of the land surface.



Q75. Consider the following statements,

  1. In a seismograph, P waves are recorded earlier than S waves.
  2. In P waves, the individual particles vibrate to and fro in the direction of wave propagation whereas in S waves, the particles vibrate up and down at right angles to the direction of wave propagation.

Which of the statements given above is /are correct?
(a) 1 only
(b) 2 only
(c) Both 1 and 2
(d) Neither 1 nor 2

Ans: C

Difficulty: Easy

Subject: Geography

Solution:

Statement I is correct: P-waves (primary waves) move faster and are the first to arrive at the surface. The P-waves are similar to sound waves. They travel through gaseous, liquid and solid materials. S-waves (Secondary waves) arrive at the surface with some time lag.

Statement II is correct: P-waves vibrate parallel to the direction of the wave. The direction of vibrations of S-waves is perpendicular to the wave direction in the vertical plane.


Source: NCERT CLASS 11 - FUNDAMENTALS OF PHYSICAL GEOGRAPHY



Q76. With reference to coal-based thermal power plants in India, consider the following statements:

  1. None of them uses seawater.
  2. None of them is set up in water-stressed districts.
  3. None of them is privately owned.

How many of the above statements are correct?
(a) Only one
(b) Only two
(c) All three
(d) None

Ans: D

Difficulty: Medium

Subject: Geography

Solution:

Statement I is not correct: In India, several coal-fired thermal power plants employ seawater for a variety of functions, including cooling the condenser system. Eg. Mundra Thermal Power Plant employs a closed-cycle induced draft circulating cooling water system that utilizes seawater.

(https://www.nsenergybusiness.com/projects/mundra-thermal-power-project/)

Statement II is not correct: New WRI research finds that 40 percent of the country’s thermal power plants are located in areas facing high water stress, a problem since these plants use water for cooling.

(https://www.wri.org/parched-power-water-demands-risks-and-opportunities-indias-power-sector)

Statement III is not correct: There are both privately and publicly owned coal-fired thermal power stations in India.

(https://cea.nic.in/wp-content/uploads/pdm/2021/06/list_power_stations_2021.pdf)



Q77. ‘Wolbachia method' is sometimes talked about with reference to which one of the following?

(a) Controlling the viral diseases spread by mosquitoes

(b) Converting crop residues into packing material

(c) Producing biodegradable plastics

(d) Producing biochar from thermo-chemical conversion of biomass

Ans: A

Difficulty: Hard

Subject: Science and Technology

Solution: This technique requires the release of a large number of male mosquitoes to reduce the overall mosquito population.

Source: http://www.eliminatedengue.com/our-
research/Wolbachia#:~:text=This%20techniq
ue%20requires%20the%20release,population%20
of%20mosquitoes%20gradually%20returns.



Q78. Consider the following activities

  1. Spreading finely ground basalt rock on farmlands extensively
  2. Increasing the alkalinity of oceans by adding lime
  3. Capturing carbon dioxide released by various industries and pumping it into abandoned subterranean mines in the form of carbonated waters

How many of the above activities are often considered and discussed for carbon capture and sequestration?
(a) Only one
(b) Only two
(c) All three
(d) None

Ans: C

Difficulty: Medium

Subject: Environment

Solution:

Statement 1 is correct: Sprinkling powdered basalt over natural ecosystems would remove vast amounts of carbon dioxide from the Earth’s atmosphere while also improving soils. That’s the finding of a new study that evaluates this proposed geoengineering scheme and estimates the costs involved.

(https://physicsworld.com/a/sprinkling-basalt-over-soil-could-remove-huge-amounts-of-carbon-dioxide-from-the-atmosphere/)

Statement 2 is correct: Scientists say they have found a workable way of reducing carbon dioxide levels in the atmosphere by adding lime to seawater. And they think it has the potential to dramatically reverse CO2 accumulation in the atmosphere

(https://www.sciencedaily.com/releases/2008/07/080721001742.htm#)

Statement 3 is correct:




Q79. 'Aerial metagenomics' best refers to which one of the following situations?

(a) Collecting DNA samples from air in a habitat at one go

(b) Understanding the genetic makeup of avian species of a habitat

(c) Using air-borne devices to collect blood samples from moving animals

(d) Sending drones to inaccessible areas to collect plant and animal samples from land surfaces and water bodies

Ans: A

Difficulty: Medium

Subject: Science and Technology

Solution: Aerial metagenomics typically refers to the study of genetic material collected from the air, usually in the form of airborne particles or aerosols. It involves analysing the microbial communities present in the air and characterising their genetic diversity.

Source: https://www.economist.com/science-and-technology/2022/01/08/wildlife-can-now-be-detected-by-sniffing-dna-in-the-air



Q80. ‘Microsatellite DNA' is used in the case of which one of the following?

(a) Studying the evolutionary relationships among various species of fauna

(b) Stimulating 'stem cells' to transform into diverse functional tissues

(c) Promoting clonal propagation of horticultural plants

(d) Assessing the efficacy of drugs by conducting series of drug trials in a population

Ans: A

Difficulty: Hard

Subject: Science and Tech

Solution:

Microsatellite, as related to genomics, refers to a short segment of DNA, usually one to six or more base pairs in length, that is repeated multiple times in succession at a particular genomic location. These DNA sequences are typically non-coding. The number of repeated segments within a microsatellite sequence often varies among people, which makes them useful as polymorphic markers for studying inheritance patterns in families or for creating a DNA fingerprint from crime scene samples. Researchers can analyse the genetic structure, population dynamics, and evolutionary relationships among various species of animals by analysing microsatellite DNA.

Source: https://www.nature.com/articles/s41598-020-77480-6



Q81. Consider the following statements in relation to Janani Suraksha Yojana:

  1. It is a safe motherhood intervention of the State Health Departments.
  2. Its objective is to reduce maternal and neonatal mortality among poor pregnant women.
  3. It aims to promote institutional delivery among poor pregnant women.
  4. Its objective includes providing public health facilities to sick infants up to one year of age.

How many of the statements given above are correct?
(a) Only one
(b) Only two
(c) Only three
(d) All four

Ans: B

Difficulty: Medium

Subject: Current Affairs

Solution: 2 and 3 are correct.

Statement 1 is not correct: It is a centrally sponsored scheme.

Statement 2 and 3 are correct: Objective of JSY: Reducing maternal and infant mortality by promoting institutional delivery among pregnant women

Statement 4 is not correct: This scheme is for pregnant women and not for infants.

Note: Under the Janani Shishu Suraksha Karyakaram scheme(and not under JSY), all pregnant women and infants will get free treatment benefits at public health institutions including zero expenses delivery.

Source: https://www.pib.gov.in/PressReleasePage.aspx?PRID=1843841



Q82. Consider the following statements in the context undertaken of interventions being under Anaemia Mukt Bharat Strategy:

  1. It provides prophylactic calcium supplementation for pre-school children, adolescents and pregnant women.
  2. It runs a campaign for delayed cord clamping at the time of child-birth.
  3. It provides for periodic deworming to children and adolescents.
  4. It addresses non-nutritional causes of anaemia in endemic pockets with special focus on malaria, hemoglobinopathies and fluorosis.

How many of the statements given above are correct?

(a) Only one

(b) Only two

(c) Only three

(d) All four

Ans: C

Difficulty: Hard

Subject: Current Affairs

Solution: Statement 1 is not correct: Statement 1 should have Iron instead of calcium. A key intervention is to give Prophylactic Iron and Folic Acid supplementation to children, adolescents and women of reproductive age and pregnant women irrespective of anemia, under Anaemia Mukt Bharat.

Statements 2, 3 and 4 are correct.




Q83. Consider the following statements'

  1. Carbon fibres are used in the manufacture of components used in automobiles and aircrafts.
  2. Carbon fibres once used cannot be recycled.

Which of the statements given above is /are correct?
(a) 1 only
(b) 2 only
(c) Both 1 and 2
(d) Neither 1 nor 2

Ans: A

Difficulty: Easy

Subject: Science and Technology

Solution:

Statement 1 is correct: Carbon fibres are widely used in modern industrial applications as they are high-strength, light in weight and more reliable than other materials. Carbon fibre is increasingly celebrated as a wonder material for the clean economy. Its unique combination of high strength and low weight has helped drive the wind power revolution and make planes more fuel efficient. Auto makers are also waking up to the material’s potential to make lighter and more efficient vehicles.

Statement 2 is not correct: Currently, the most common recycling process used to recover carbon fiber from composite waste is pyrolysis, where high heat basically burns off the resin. Solvolysis, which uses a solvent to dissolve the resin, has long been claimed to offer superior properties. So far, commercialization of both pyrolysis and solvolysis has been through batch processes. But inline processing is finally in the works.

(https://www.compositesworld.com/articles/sustainable-inline-recycling-of-carbon-fiber)



Q84. Consider the following actions:

  1. Detection of car crash/collision which results in the deployment of airbags almost instantaneously
  2. Detection of accidental free fall of a laptop towards the ground which results in the immediate turning off of the hard drive
  3. Detection of the tilt of the smartphone which results in the rotation of display between portrait and landscape mode

In how many of the above actions is the function of the accelerometer required?

(a) Only one

(b) Only two

(c) All three

(d) None

Ans: C

Difficulty: Medium

Subject: Science and Technology

Solution: An accelerometer is an electronic sensor that measures the acceleration forces acting on an object, in order to determine the object’s position in space and monitor the object’s movement.

  1. In the case of a car crash/collision, the accelerometer is used to detect the sudden deceleration of the vehicle and trigger the deployment of the airbags.

(https://blog.st.com/bringing-car-crash-detection-to-personal-electronics/)

  1. In the case of a laptop falling, the accelerometer is used to detect the sudden change in motion and trigger the immediate turning off of the hard drive to prevent damage to the data.

(https://www.analog.com/en/analog-dialogue/articles/using-accelerometers-to-protect-hard-drives.html)

  1. An accelerometer is also required for the detection of the tilt of the smartphone which results in the rotation of the display between portrait and landscape mode. Note: Gyroscope is also used while rotating the phone

(https://www.scientificamerican.com/article/science-with-a-smartphone-accelerometer/)



Q85. With reference to the role of biofilters in Recirculating Aquaculture System, consider the following statements:

  1. Biofilters provide waste treatment by removing uneaten fish feed.
  2. Biofilters convert ammonia present in fish waste to nitrate.
  3. Biofilters increase phosphorus as nutrient for fish in water.

How many of the statements given above are correct?
(a) Only one
(b) Only two
(c) All three
(d) None

Ans: B

Difficulty: Hard

Subject: Science and Technology

Solution: 1 and 2 are correct, 3 is incorrect.

Biofilters are designed to remove pathogens from recycled solutions by filtration and by exposing these pathogens for extended periods of time to unfavourable conditions reducing their viable population

Statement 1 is correct: A filtering (biofilter) system is necessary to purify the water and remove or detoxify harmful waste products and uneaten feed. Biofilters use microorganisms, which are capable of degrading many compounds, fixed to an inorganic/organic medium (carrier) to break down pollutants present in a fluid stream. They also provide waste treatment by removing uneaten fish feed (see image attached below)

(https://fisheries.tamu.edu/files/2013/09/Fish-Farming-in-Recirculating-Aquaculture-Systems-RAS.pdf)

Statement 2 is correct: A properly designed biofilter will keep ammonia and nitrite concentrations at levels that are not harmful to the fish. Biofilters convert ammonia nitrogen to nitrite nitrogen and then to the less toxic nitrate nitrogen for the fish.

(https://www.sciencedirect.com/topics/agricultural-and-biological-sciences/biofilter)

Statement 3 is not correct: Phosphorus (P) is removed to reduce the potential for nutrient enrichment in waters.



Q86. Consider the following pairs :
Objects in space    Description
1. Cepheids : Giant clouds of dust and gas in space
2. Nebulae : Stars which brighten and dim periodically
3. Pulsars : Neutron stars that are formed when massive stars run out of fuel and collapse

How many  of the above pairs are correctly matched?
(a) Only one
(b) Only two
(c)  All three
(d) None

Ans: A

Difficulty: Medium

Subject: Science and Tech

Solution: Only the third pair is matched correctly. Correct pairs are:

Cepheids - Stars which brighten and dim periodically

Nebulae - Giant clouds of dust and gas in space

Pulsars - Neutron stars that are formed when massive stars run out of fuel and collapse

Source:

  1. https://www.space.com/15396-variable-stars.html
  2. https://spaceplace.nasa.gov/nebula/en/#
  3. https://imagine.gsfc.nasa.gov/science/objects/neutron_stars1.html#


Q87. Which one of the following countries has its own Satellite Navigation System?
(a) Australia
(b) Canada
(c) Israel
(d) Japan

Ans: D

Difficulty: Easy

Subject: Science and Technology

Solution: QZSS is a Japanese satellite positioning system composed mainly of satellites in quasi-zenith orbits (QZO)

Source: https://www.gps.gov/systems/gnss/



Q88. Consider the following statements

  1. Ballistic missiles are jet-propelled at subsonic speeds throughout their flights, while cruise missiles are rocket-powered only in the initial phase of flight.
  2. Agni-V is a medium-range supersonic cruise missile, while BrahMos is a solid-fuelled intercontinental ballistic missile.

Which of the statements given above is /are correct?
(a) 1 only
(b) 2 only
(c) Both 1 and 2
(d) Neither 1 nor 2

Ans: D

Difficulty: Easy

Subject: Science and Technology

Solution:

Statement 1 is not correct: Cruise missiles are jet-propelled at subsonic speeds throughout their flights, while ballistic missiles are rocket-powered only in the initial phase of flight. (cruise and ballistic are interchanged).

Statement 2 is not correct: The Agni-V is an Indian intercontinental ballistic missile (ICBM) with a range of over 5,000 km. BrahMos is a universal long-range supersonic cruise missile

Source: https://www.timesnownews.com/india/agni-to-brahmos-the-indian-missiles-than-can-disintegrate-any-hostile-rivals-article-96281501#



Q89. Consider the following statements regarding mercury pollution

  1. Gold mining activity is a source of mercury pollution in the world.
  2. Coal-based thermal power plants cause mercury pollution.
  3. There is no known safe level of exposure to mercury.

How many of the above statements are correct?
(a) Only one
(b) Only two
(c) All three
(d) None

Ans: C

Difficulty: Hard

Subject: Environment

Solution:

Statement 1 and 2 are correct: Globally, artisanal and small-scale gold mining (ASGM) is the largest source of anthropogenic mercury emissions (37.7%), followed by stationary combustion of coal (21%). Other large sources of emissions are non-ferrous metals production  (15%) and cement production (11%). (United Nations Environment Programme, Global Mercury Assessment, 2018)

(https://www.epa.gov/international-cooperation/mercury-emissions-global-context)

Statement 3 is correct:

(Source: https://www.unep.org/news-and-stories/press-release/world-comes-together-tackle-mercury-poisoning)



Q90. With reference to green hydrogen, consider the following statements:

  1. It can be used directly as a fuel for internal combustion.
  2. It can be blended with natural gas and used as fuel for heat or power generation.
  3. It can be used in the hydrogen fuel cell to run vehicles.

How many of the above statements are correct?
(a) Only one
(b) Only two
(c) All three
(d) None

Ans: C

Difficulty: Easy

Subject: Science and Technology

Solution:

What is green hydrogen?

Green hydrogen is defined as hydrogen produced by splitting water into hydrogen and oxygen using renewable electricity.

Statement 1 is correct: Hydrogen engines (HCE) burn hydrogen in an internal combustion engine, in just the same way gasoline is used in an engine. In general, getting an internal combustion engine to run on hydrogen is not difficult.

(https://www1.eere.energy.gov/hydrogenandfuelcells/tech_validation/pdfs/fcm03r0.pdf)

Statement 2 is correct: Hydrogen and CNG blends (HCNG) may be considered as an automotive fuel without any major modification in the existing CNG engine and infrastructure.

(https://www.sae.org/publications/technical-papers/content/2007-01-2045/)

Statement 3 is correct: Fuel cell electric vehicles (FCEVs) are powered by hydrogen. They are more efficient than conventional internal combustion engine vehicles and produce no harmful tailpipe emissions—they only emit water vapor and warm air.

(https://afdc.energy.gov/vehicles/fuel_cell.html)

Source:



Q91. Consider the following statements with reference to India:

  1. According to the Micro, Small and Medium Enterprises Development (MSMED) Act, 2006', the 'medium enterprises' are those with investments in plant and machinery between ₹15 crore and ₹25 сrore.
  2. All bank loans to the Micro, Small and Medium Enterprises qualify under the priority sector.

Which of the statements given above is /are correct?
(a) 1 only
(b) 2 only
(c) Both 1 and 2
(d) Neither 1 nor 2

Ans: D

Difficulty: Easy

Subject: Economy

Solution:

Statement 1 is not correct:

Classification of MSMEs

(i) a micro enterprise, where the investment in plant and machinery does not exceed twenty- five lakh rupees;

(ii) a small enterprise, where the investment in plant and machinery is more than twenty-five lakh rupees but does not exceed five crore rupees; or

(iii) a medium enterprise, where the investment in plant and machinery is more than five crore rupees but does not exceed ten crore rupees;

Statement 2 is not correct:

Only loans given to MSMEs conforming to certain guidelines is classified as a priority sector lending (https://www.rbi.org.in/Scripts/NotificationUser.aspx?Id=11959&Mode=0#MSME)

(Source: RBI Website)


Q92. With reference to Central Bank digital currencies, consider the following statements:

  1. It is possible to make payments in a digital currency without using the US dollar or SWIFT system.
  2. A digital currency can be distributed with a condition programmed into it such as a timeframe for spending it.

Which of the statements given above is /are correct?
(a) 1 only
(b) 2 only
(c) Both 1 and 2
(d) Neither 1 nor 2

Ans: C

Difficulty: Medium

Subject: Economy

Solution: Central bank digital currencies (CBDCs) are a form of digital currency issued by a country's central bank. They are similar to cryptocurrencies, except that their value is fixed by the central bank and equivalent to the country's fiat currency.

Statement 1 is correct: Using CBDC ountries will be able to directly exchange digital currencies in a bilateral way and without going through SWIFT or similar settlement systems.

(https://www.brookings.edu/techstream/the-innovators-dilemma-and-u-s-adoption-of-a-digital-dollar/)

Statement 2 is correct: A digital currency can be distributed with a condition programmed into it such as a timeframe for spending it.

Eg. The digital yuan is programmable to the point that the currency can be made to expire, thus forcing consumers to use it up by a certain date

(https://bfsi.economictimes.indiatimes.com/news/policy/digital-currency-yuan-comes-with-an-expiry-date-spend-or-it-will-vanish/82059471)



Q93. In the context of finance, the term ‘beta' refers to

(a) the process of simultaneous buying and selling of an asset from different platforms

(b) an investment strategy of a portfolio manager to balance risk versus reward

(c) a type of systemic risk that arises where perfect hedging is not possible

(d) a numeric value that measures the fluctuations of a stock to changes in the overall stock market

Ans: D

Difficulty: Medium

Subject: Economy

Solution:

Beta (β) is a measure of the volatilityor systematic riskof a security or portfolio compared to the market as a whole (usually Index such as NIFTY or SENSEX). Stocks with betas higher than 1.0 can be interpreted as more volatile than the index.

Source: https://www.investopedia.com/terms/b/beta.asp



Q94. Consider the following statements

  1. The Self-Help Group (SHG) programme was originally initiated by the State Bank of India by providing microcredit to the financially deprived.
  2. In an SHG, all members of a group take responsibility for a loan that an individual member takes.
  3. The Regional Rural Banks and Scheduled Commercial Banks support SHGs.

How many of the above statements are correct?
(a) Only one
(b) Only two
(c) All three
(d) None

Ans: B

Difficulty: Easy

Subject: Economy

Solution:

Statement 1 is correct: History of SHGs in India-

In India, banks are the predominant agency for delivery of micro-credit. In 1970, Ilaben Bhat, founder member of ‘SEWA’ (Self Employed Women’s Association) in Ahmadabad, had developed a concept of ‘women and micro-finance’. The Annapurna Mahila Mandal’ in Maharashtra and ‘Working Women’s Forum’ in Tamilnadu and many National Bank for Agriculture and Rural Development (NABARD)-sponsored groups have followed the path laid down by ‘SEWA’. ‘SEWA’ is a trade union of poor, self-employed women workers.

Since 1987 ‘Mysore Resettlement and Development Agency’ (MYRADA) has promoted Credit Management Groups (CMGs). CMGs are similar to self-help groups. The basic features of this concept promoted by MYRADA are: 1] Affinity, 2] Voluntarism, 3] Homogeneity and 4] Membership should be limited to 15-20 persons. Aim of the CMG is to bestow social empowerment to women.

In 1991-92 NABARD started promoting self-help groups on a large scale. And it was the real take-off point for the ‘SHG movement’. In 1993, the Reserve Bank of India also allowed SHGs to open saving accounts in banks. Facility of availing bank services was a major boost to the movement.

(Source: https://ncwapps.nic.in/pdfReports/SHG-Maharashtra.pdf)

Statement 2 is correct: Joint Liability Groups (JLGs) is a mechanism through which  all members of a self help group take responsibility for a loan that an individual member takes.

Statement 3 is correct: The Regional Rural Banks and Scheduled Commercial Banks support SHGs by giving loans to them. These loans are then accounted for fulfilling the priority sector requirements of RRBs and commercial banks.

(http://pggc46.ac.in/images/RRBS.pdf)



Q95. Consider the following statements:
Statement-I: India's public sector health care system largely focuses on curative care with limited preventive, promotive and rehabilitative care.

Statement-II: Under India's decentralized approach to health care delivery, the States are primarily responsible for organizing health services.

Which one of the following is correct in respect of the above statements?

(a) Both Statement-I and Statement-II are correct and Statement-II is the correct explanation for Statement-I

(b) Both Statement-I and Statement-II are correct and Statement-II is not the correct explanation for Statement-I

(c) Statement-I is correct but Statement-II is incorrect

(d) Statement-I is incorrect but Statement-II is correct

Ans: B

Difficulty: Medium

Subject: Economy

Solution:

Statement I is correct: Over 50% of total health expenditure on India is on curative care. Whereas expenditure on preventive, promotive and rehabilitative care is less than 10%

National Health Accounts 2019-20 (page 20)

https://nhsrcindia.org/sites/default/files/2023-04/National%20Health%20Accounts-2019-20.pdf

Controversial Question: According to data, statement I becomes wrong, but going by National Health Policy, 2017, It becomes correct.

As per the National Health Policy, 2017, India’s public sector health care system focuses on improving health status through coordinated policy action in all sectors. It also focuses on expanding preventive, promotive, curative, palliative and rehabilitative services through the public health sector which focuses on quality of these services.

Statement II is correct:

Source:



Q96. Consider the following statements:
Statement-1: According to the ‘United Nations World Water Development Report, 2022'. India extracts more than a quarter of the world's groundwater withdrawal each year.

Statement-II: India needs to extract more than a quarter of the world's groundwater each year to satisfy the drinking water and sanitation needs of almost 18% of the world's population living in its territory.

Which one of the following is correct in respect of the above statements?

(a) Both Statement-1 and Statement-II are correct and Statement-II is the correct explanation for Statement-I

(b) Both Statement-I and Statement-II are correct and Statement-II is not the correct explanation for Statement-I

(c) Statement-I is correct but Statement-II is incorrect

(d) Statement-I is incorrect but Statement-II is correct

Ans: C

Difficulty: Easy

Subject: Environment

Solution: According to the United Nations World Water Development Report, 2022, India extracts around 26% of the world’s total groundwater withdrawal. So, statement 1 is correct

However, 89% of the groundwater extracted in India is used for agricultural irrigation purposes. (Table 5.1 in United Nations World Water Development Report, 2022). Thus, statement 2 is not correct.




Q97. Consider the following statements:

  1. According to the Constitution of India, the Central Government has a duty to protect States from internal disturbances.
  2. The Constitution of India exempts the States from providing legal counsel to a person being held for preventive detention.
  3. According to the Prevention of Terrorism Act, 2002, confession of the accused before the police cannot be used as evidence.

How many of the above statements are correct?
(a) Only one
(b) Only two
(c) All three
(d) None

Ans: A

Difficulty: Hard

Subject: Polity

Solution: Statement 1 is correct. 2 and 3 are not correct.

Controversial Question: Statement 2 is not verbatim given in the constitution. But, if read with NALSA judgement in mind, it leads to confusion in this statement. When UPSC asks questions about what is given in the constitution, it usually considers verbatim statements (observation from PYQs).

Statement 1 is correct: Article 355 in The Constitution Of India: Duty of the Union to protect States against external aggression and internal disturbance It shall be the duty of the Union to protect every State against external aggression and internal disturbance and to ensure that the government of every State is carried on in accordance with the provisions of this Constitution. (https://indiankanoon.org/doc/490234/)

Statement 2 is not correct: Article 22 of the Indian constitution deals with the rights of a person under arrest or detention. There is no provision in this article which is directly mentioned in Article 22. So, statement 2 becomes incorrect.

Source: https://indiankanoon.org/doc/581566/



Q98. Which one of the following countries has been suffering from decades of civil strife and food shortages and was in news in the recent past for its very severe famine?

(a) Angola

(b) Costa Rica

(c) Ecuador

(d) Somalia

Ans: D

Difficulty: Medium

Subject: Current Affairs

Solution: A devastating drought, coupled with the effects of three decades of conflict, have created a catastrophic food crisis.

Source:https://www.icrc.org/en/document/somalia-food-crisis-drought-conflict-and-fight-survival



Q99. Consider the following statements:

  1. In India, the Biodiversity Management Committees are key to the realization of the objectives of the Nagoya Protocol.
  2. The Biodiversity Management Committees have important functions in determining access and benefit sharing, including the power to levy collection fees on the access of biological resources within its jurisdiction.

Which of the statements given above is /are correct?
(a) 1 only
(b) 2 only
(c) Both 1 and 2
(d) Neither 1 nor 2

Ans: C

Difficulty: Easy

Subject: Environment

Solution: Statement 1 is correct Biodiversity Management Committees are key to the realisation of the objectives of the Nagoya Protocol (2010), negotiated within the CBD, enjoining parties to take measures to ensure that benefits from the utilisation of genetic resources and associated traditional knowledge accrue to indigenous and local communities on mutually agreed terms.

Statement 2 is correct:

(https://www.indiacode.nic.in/bitstream/123456789/2046/1/200318.pdf)

Source: https://sbb.haryanaforest.gov.in/project/biodiversity-management-committee-bmc/#:~:text=The%20main%20function%20of%20the,traditional%20knowledge%20associated%20with%20them.


Q100. Consider the following statements in respect of election to the President of India:

  1. The members nominated to either House of the Parliament or the Legislative Assemblies of States are also eligible to be included in the Electoral College.
  2. Higher the number of elective Assembly seats, higher is the value of vote of each MLA of that State.
  3. The value of vote of each MLA of Madhya Pradesh is greater than that of Kerala.
  4. The value of vote of each MLA of Puducherry is higher than that of Arunachal Pradesh because the ratio of total population to total number of elective seats in Puducherry is greater as compared to Arunachal Pradesh.

How many of the above statements are correct?
(a) Only one
(b) Only two
(c) Only three
(d) All four

Ans: A

Difficulty: Hard

Subject: Polity

Solution:

Statement 1 is not correct: only elected members (and not nominated) are eligible to be included in the electoral college of the Election of President in India.

Article 54 of Constitution of India: Election of President The President shall be elected by the members of an electoral college consisting of the elected members of both Houses of Parliament; and the elected members of the Legislative Assemblies of the States

(https://indiankanoon.org/doc/887242/)

Statement 2 is not correct: Higher the number of elective Assembly seats, lower is the value of vote of each MLA of that State.

Statement 3 and 4 are not correct: Refer following table.


What Next?


The result of UPSC Prelims will be out by 15th to 20th June 2023.

If you scoring anything above 75, you should start your mains preparation.

This is because in the past, there have been issues with Prelims papers, with UPSC cancelling some questions at the time of the release of the official key. This happened in 2020, and is likely to happen this year as well.

Till results, focus on

  1. Optional
  2. Mains Specific GS Subjects
  3. Writing 2 answers/day

If you are want to streamline your preparation with personalised mentor support, check out our courses.